In a poll of 510 human resource​ professionals, ​%45.9% said that body piercings and tattoos were big personal grooming red flags. Complete parts​ (a) through​ (d) below.

Answers

Answer 1

By sample distribution, (0.431,0.487) is Complete parts​ (a) through​ (d) below.

What does a statistical sample distribution mean?

An accurate probability distribution of a statistic is known as a sampling distribution, and it is created by repeatedly sampling a certain population. It depicts a spectrum of potential results for a statistic, such as the mean or mode of a variable, for a population.

Given

Favourable cases (X)=?

Sample size(n)=510

Sample proportion (p_hat)=X/n=0.459

X=np_hat=510*0.459=234.09=234

Confidence level(1-α)=99%=0.99

α=0.0

The critical value for  α=0.01 is Zc=2.576

The margin of error is E=Zc* √((p_hat(1-p_hat)/n))

E=2.576* √((0.459(1-0.459)/510))=0.0568

The 99% confidence interval for p

CI=(p_hat-E,p_hat+E)

CI=(0.459-0.0568,0.458+0.0568)

CI=(0.402,0.516)

Confidence level(1-α)=80%=0.80

α=0.20

The critical value for  α=0.2 is Zc=1.282

The margin of error is E=Zc* √((p_hat(1-p_hat)/n))

E=1.282* √((0.459(1-0.459)/510))=0.0283

The 80 % confidence interval for p

CI=(p_hat-E,p_hat+E)

CI=(0.459-0.0283,0.458+0.0283)

CI=(0.431,0.487)

Learn more about sample distribution

brainly.com/question/13501743

#SPJ1


Related Questions

the widget store owner tells you that 200 customers arrive and purchase a widget from the store each day. assuming you must sell 30 of your widgets to cover the transportation costs and given the probability 0.1667, use a binomial distribution of at least covering the transportation costs (that is the probability of selling at least 30 widgets). write your answer as a probability (not a percent) rounded to 4 decimals.

Answers

Note that the probability of selling at least 30 widgets is 0.0468.

How is this so?

Given the information provided, the problem requires us to use a binomial distribution to estimate the probability of selling at least 30 widgets.

A binomial distribution is a probability distribution that describes the number of successes in a set number of trials.

Where n = 200 (the number of people that visit the store and buy a widget each day) and

p = 0.1667 (likelihood of a client purchasing a widget)

We are looking for the probability of selling at least 30 widgets, which is equivalent to finding the probability of selling 30 or more widgets.

This can be calculated using the cumulative probability formula for a binomial distribution:

P(X >= 30) = 1 - P(X < 30) = 1 - Σ (nCx)(p^x)(1-p)^(n-x) from x=0 to 29

We can use a calculator or a software to calculate this probability. The result is 0.0468 rounded to 4 decimals.

Learn more about Binomial Distribution Theorem:

brainly.com/question/29283294

#SPJ1

Someone please help I have no idea what I’m doing

Answers

The compositions are:

f * f(x) = x⁴ - 2x²

g * g(x) = 81/64x

How did we get the values?

To find the composition f * f, we start by substituting f(x) into f(x) wherever we see x:

f * f (x) = f(f(x)) = f(x² - 1) = (x² - 1)² - 1

Expanding the expression, we get:

f * f (x) = x⁴ - 2x² + 1 - 1 = x⁴ - 2x²

So, f * f(x) = x⁴ - 2x².

To find the composition g * g, we substitute g(x) into g(x) wherever we see x:

g * g (x) = g(g(x)) = g(9/8x) = 9/8(9/8x) = 81/64x

So, g * g(x) = 81/64x.

Therefore, the compositions are:

f * f(x) = x⁴ - 2x²

g * g(x) = 81/64x

learn more about composing function: https://brainly.com/question/10687170

#SPJ1

The text format of the question is:

Suppose that the functions f and g are defined as follows.

f(x) = x² - 1

g(x) = 9/8x, x ≠

Find the compositions f * f and g * g

Simplify your answers as much as possible.

(Assume that your expressions are defined for all x in the domain of the composition. You do not have to indicate the domain)

A woman has a total of $7,000 to invest. She invests part of the money in an account that pays 5%
per year and the rest in an account that pays 12% per year. If the interest earned in the first year is
$560, how much did she invest in each account?

Answers

The woman invested $4,000 at 5% and $3,000 at 12%.

To solve this problem

Let x be the amount invested at 5% and y be the amount invested at 12%. Then we have:

x + y = 7,000 (equation 1)

0.05x + 0.12y = 560 (equation 2)

To solve for x and y, we can use substitution method :

From equation 1, we have y = 7,000 - x. Substituting this into equation 2, we get:

0.05x + 0.12(7,000 - x) = 560

Simplifying and solving for x, we get:

0.05x + 840 - 0.12x = 560

-0.07x = -280

x = 4,000

Substituting x = 4,000 into equation 1, we get:

4,000 + y = 7,000

y = 3,000

Therefore, the woman invested $4,000 at 5% and $3,000 at 12%.

Learn more about substitution here: brainly.com/question/26094713

#SPJ1

A study of a country's colleges and universities resulted in the demand equation q = 20,000 − 2p, where q is the enrollment at a public college or university and p is the average annual tuition (plus fees) it charges.† Officials at Enormous State University have developed a policy whereby the number of students it will accept per year at a tuition level of p dollars is given by q = 8,500 + 0.5p. Find the equilibrium tuition price p and the consumers' and producers' surpluses at this tuition level. What is the total social gain at the equilibrium price? HINT [See Example 3.]

Answers

The equilibrium tuition price p and the consumers' and producers' surpluses at this tuition level is 11400

Find the equilibrium tuition price p and the consumers' and producers' surpluses at this tuition level

From the question, we have the following parameters that can be used in our computation:

q = 20,000 − 2p

q = 8,500 + 0.5p

The equilibrium tuition price p and the consumers' and producers' surpluses at this tuition level is when q = q

So, we have

20,000 − 2p = 8,500 + 0.5p

Evaluate the like terms

2.5p = 28500

Divide by 2.5

p = 28500/2.5

Evaluate

p = 11400

Hence, the equilibrium tuition price p is 11400

Read more about equilibrium price at

https://brainly.com/question/18426868

#SPJ1

its for 100 points please

Answers

Answer: The first choice.

Step-by-step explanation:

As you can see in the diagram, the line passes through the points[tex](x, y) = (4, -4)[/tex] and [tex](x, y) = (-4, 8)[/tex] . Thus, the slope of the line is [tex]\frac{8-(-4)}{-4-(4)} = \frac{12}{-8} = -\frac{3}{2}.[/tex] Thus, the correct answer is the first choice, as that line has a slope of [tex]-\frac64=-\frac32.[/tex]


Which graph best represents a line that passes
through the values in the table below?
X. Y
0. 1
1. -2
2. - 5

Answers

Answer:

y = -3x + 1

Step-by-step explanation:

We are given a couple ordered pairs from this table. Lets use the slope formula so we can find the slope of this line:

[tex]\frac{y_2-y_1}{x_2-x_1}[/tex]

Y2, y1, x2, and x1 are constants (points on the line). To make it easy, lets take the coordinates (0,1) and (1,-2). Then we just plug these points in and evaluate.

[tex]\frac{-2-1}{1-0}[/tex]

Evaluate both sides of the fraction.

[tex]\frac{-3}{1} =-3[/tex]

Now that we have the slope, we are given the y intercept coordinate, (0,1) which means the y-intercept is 1. To write it in y = mx + b form, we just plug in our numbers and were good to go.

[tex]y=-3x+1[/tex]

Estimate how much a paramedic will make over 30 years

Answers

Answer:

The salary of a paramedic can vary depending on several factors such as location, experience, and employer. According to the Bureau of Labor Statistics, the median annual wage for paramedics and EMTs was $36,650 in May 2020.

Assuming a constant annual income of $36,650 over 30 years, the estimated total income for a paramedic would be:

$36,650 x 30 = $1,099,500

However, it is important to note that this is just an estimate and does not take into account potential salary increases, promotions, or changes in the job market.

If apples cost $3.12 per pound and you have a bag that weighs 3pounds and 8 ounces, how much will you pay for those apples?

Answers

Answer: $10.92

Step-by-step explanation:

Since there are 16 ounces in a pound, 8 ounces would be equivalent to half a pound. Thus, 3 pounds and 8 ounces = 3.5 pounds.

Since apples cost $3.12 per pound and we have 3.5 pounds, the total cost would be $3.12*3.5 pounds = $10.92

You visit an aquarium. One of the tanks at the aquarium holds 450 gallons of water. What should the third dimension be so that the diagram shows one possible set of dimensions of the tank?
(1 gal =
231 in.3
)



With a width of 21 inches and a height of 33 inches, the length needs to be
inches.

Answers

The length of the tank should be approximately 150.4 inches in order to hold 450 gallons of water, with a width of 21 inches and a height of 33 inches.

How to determine the length of the tank

To find the length of the tank, we first need to convert the volume of water from gallons to cubic inches, using the given conversion factor:

1 gallon = 231 cubic inches

Therefore, the volume of water in the tank is:

450 gallons * 231 cubic inches/gallon = 103950 cubic inches

We can then use the formula for the volume of a rectangular prism to find the length of the tank:

Volume = length * width * height

Substituting the given dimensions, and solving for the length, we get:

103950 cubic inches = length * 21 inches * 33 inches

length = 103950 cubic inches / (21 inches * 33 inches)

length ≈ 150.4 inches

Therefore, the length of the tank should be approximately 150.4 inches in order to hold 450 gallons of water, with a width of 21 inches and a height of 33 inches.

Learn more about length at https://brainly.com/question/28322552

#SPJ1

The relationship between the actual air temperature x (in
degrees Fahrenheit) and the temperature y adjusted for wind
chill (in degrees Fahrenheit, given a 30 mph wind) is given by
the following formula:

y = -26 +1.3x

Estimate the actual temperature if the temperature adjusted
for wind chill is -35 degrees Fahrenheit.

Answers

The estimated actual temperature (x) when the temperature adjusted for wind chill (y) is -35 degrees Fahrenheit =

approximately -6.92 degrees Fahrenheit.

How do we estimate the actual temperature?

To estimate the actual temperature (x) when the temperature adjusted for wind chill (y) is given as -35 degrees Fahrenheit using the formula y = -26 + 1.3x, we can substitute y with -35 and solve for x.

-35 = -26 + 1.3x

Adding 26 to both sides to isolate the x term:

-35 + 26 = 1.3x

-9 = 1.3x

Dividing both sides by 1.3 to solve for x:

x = -9 / 1.3

x ≈ -6.92

Thus, the estimated actual temperature (x) when the temperature adjusted for wind chill (y) is -35 degrees Fahrenheit would be approximately -6.92 degrees Fahrenheit.

Learn more about temperature at brainly.com/question/27944554

#SPJ1

The estimated actual temperature is -6.92 degrees Fahrenheit.

The estimated temperature

We can use the given formula to solve for x when y is known. Rearranging the formula, we get:

x = (y + 26) / 1.3

Substituting y = -35 into this formula, we get:

x = (-35 + 26) / 1.3

x = -6.92 (rounded to two decimal places)

Therefore, the estimated actual temperature is -6.92 degrees Fahrenheit.

The term actual temperature typically refers to the true or measured temperature of an object, substance, or environment. It is the temperature that is currently present in a given location or situation, as opposed to a predicted or estimated temperature. The actual temperature can be measured using a thermometer or other temperature sensing device.

Learn more on calculations on temperature here https://brainly.com/question/25677592

#SPJ1

24. Which description describes the solutions to this inequality?
9 > 52
A. all the possible numbers that are smaller than twenty-five
B. all the positive numbers that are greater than ten
C. all the positive numbers that are larger than twenty-five
D. all the possible numbers that are smaller than ten

Answers

The description describes the solutions to this inequality in all the positive numbers that are greater than ten. The correct option is B.

What is inequality?

Relationships between two expressions that aren't equal to one another are known as inequalities. Social justice theories are fundamentally based on the idea of inequality, which is the condition of not being equal, especially in terms of status, rights, and opportunities.

The given equation is 9 > 52.

everything that is positive and bigger than 10

Therefore, the correct option is B. All the positive numbers are greater than ten.

Learn more about inequality, here:

https://brainly.com/question/30238773

#SPJ1

a senior math contest began at 6:00 am on a Sunday and ended 1000 min later then did the contest get over

Answers

The senior math contest ended at 10:40 pm on Sunday

What time did the contest get over

If the senior math contest began at 6:00 am on a Sunday and ended 1000 minutes later, we can add 1000 minutes to the starting time to find the ending time.

To do this, we can first convert the starting time of 6:00 am into minutes. There are 60 minutes in one hour, so 6:00 am is equivalent to:

6 hours × 60 minutes/hour = 360 minutes

Now, we can add 1000 minutes to this starting time:

360 minutes + 1000 minutes = 1360 minutes

Finally, we can convert this total number of minutes back into hours and minutes.

Since there are 60 minutes in one hour, we can divide 1360 minutes by 60 to get:

1360 minutes ÷ 60 = 22 hours and 40 minutes

Therefore, the senior math contest ended at 10:40 pm on the day (Sunday).

Read more about expression at

https://brainly.com/question/15775046

#SPJ1

During a sale the regular price of a pair of running shoes is reduced by 20%. If the sales price is $64 what is the regular price of the running shoes

Answers

The regular price of the running shoes is $80.

Let's denote the regular price of the running shoes as P. We know that during the sale, the price is reduced by 20%, which means the sales price is 80% (100% - 20%) of the regular price.

Given that the sales price is $64, we can set up the equation:

0.8P = $64

To find the regular price P, we divide both sides of the equation by 0.8:

P = $64 / 0.8

P = $80

For more such questions on price

https://brainly.com/question/20475128

#SPJ8

population of a certain inner-city area is estimated to be declining according to the model P(1) = 108,000€-0.011, where t is the number of years from the present What does this model predict the population will be in 7 years? Round to the nearest person.

Answers

The model predicts that the population of the inner-city area will be 107,079 people in 7 years

How is the population of an area or country determined?

The number of inhabitants in a given region is characterized as the quantity of individuals normally living around there, estimated on 1 January in a given year.

The given model for the number of inhabitants in the rough part of town region is:

P(t) = 108,000 - 0.011t

where t is the number of years from the present and P(t) is the estimated population at time t.

To find the predicted population in 7 years, we can substitute t=7 into the given model and evaluate:

P(7) = 108,000 - 0.011(7)

P(7) = 108,000 - 0.077

P(7) = 107,923

As a result, the model projects that the inner-city area will have a population of 107,079 people in seven years, rounded to the nearest person.

know more about area visit :

https://brainly.com/question/27683633

#SPJ1

Which classification best represents a triangle with side lengths 6 cm, 10 cm, and 12 cm?

acute, because 62 + 102 < 122
acute, because 6 + 10 > 12
obtuse, because 62 + 102 < 122
obtuse, because 6 + 10 > 12

Answers

Answer:

To determine the classification of a triangle based on its side lengths, we can use the Pythagorean theorem, which states that in a right triangle, the sum of the squares of the lengths of the legs (the shorter sides) is equal to the square of the length of the hypotenuse (the longest side).

For this particular triangle with side lengths 6 cm, 10 cm, and 12 cm, we can apply the Pythagorean theorem to determine whether the triangle is acute (all angles are less than 90 degrees), obtuse (one angle is greater than 90 degrees), or right (one angle is equal to 90 degrees).

Using the Pythagorean theorem, we find:

6^2 + 10^2 = 136

12^2 = 144

Since 136 is less than 144, we know that 6 cm, 10 cm, and 12 cm are the side lengths of a triangle that is obtuse (one angle is greater than 90 degrees), because the longest side (the hypotenuse) is greater than the sum of the squares of the lengths of the other two sides.

Therefore, the correct answer is: obtuse, because 6^2 + 10^2 < 12^2.

Owen has earned 19 out of the 50 points for his service
grade. He must submit the rest of these points by the last
day of the grading period. How many more points must he
earn to meet his goal?

Answers

The number of more points that Owen must earn to meet his goal would be = 31 points.

How to calculate the extra points needed by Owen?

The total number of point expected to be scored by Owen = 50 points

The total amount of points that Owen has earned so far = 19 point.

The amount of points that Owen should earn more to meet up with the points expected of him = 50-19= 31 points.

Learn more about subtraction here:

https://brainly.com/question/25421984

#SPJ1

Find the circumference

Answers

Answer:

25.12 m (or 25.13 m)

Step-by-step explanation:

Recall the formula for the circumference of a circle:

[tex]2\pi r[/tex]

where r is the radius.

We are given that the radius is 4 m, so let's plug that into our formula:

[tex]2\pi r=\\2\pi *4=\\8\pi[/tex]

[tex]\pi[/tex] is approximately 3.14, so let's multiply 8 m by 3.14. We get:

25.12

So, the circumference is about 25.12 m.

(if you multiply by pi itself using a calculator, the answer is about 25.13 m)

If you invest 40% of your money in A and 60% in B, what is the expected return and standard deviation of your portfolio?

Answers

Answer:

3.15%

Step-by-step explanation:

To find the expected return and standard deviation of a portfolio that is invested in two assets A and B, we need to know the expected returns and standard deviations of those assets, as well as the correlation between them.

Let's assume that the expected return and standard deviation of asset A are 10% and 5%, respectively, and that the expected return and standard deviation of asset B are 8% and 3%, respectively. Let's also assume that the correlation between the two assets is 0.5.

To find the expected return of the portfolio, we can use the following formula:

Expected return of portfolio = weight of A * expected return of A + weight of B * expected return of B

where the weights are the proportions of the portfolio invested in each asset:

weight of A = 40% = 0.4

weight of B = 60% = 0.6

Expected return of portfolio = 0.4 * 10% + 0.6 * 8%

Expected return of portfolio = 9.2%

Therefore, the expected return of the portfolio is 9.2%.

To find the standard deviation of the portfolio, we can use the following formula:

Standard deviation of portfolio = sqrt(weight of A^2 * variance of A + weight of B^2 * variance of B + 2 * weight of A * weight of B * correlation * standard deviation of A * standard deviation of B)

where the variances are the square of the standard deviations of each asset:

variance of A = (5%)^2 = 0.0025

variance of B = (3%)^2 = 0.0009

Standard deviation of portfolio = sqrt(0.4^2 * 0.0025 + 0.6^2 * 0.0009 + 2 * 0.4 * 0.6 * 0.5 * 0.05 * 0.03)

Standard deviation of portfolio = 0.0315 or 3.15% (rounded to two decimal places)

Therefore, the standard deviation of the portfolio is 3.15%.

Can you mark me asbrainliest if you appreciate my effort just for favor. Thank you^^

1. Mona wants to run more than 60
miles this week. She ran a total of
24 miles during the first 4 days of the
week. Which number line represents
the number of miles Mona must
run each day if she wants to meet
her goal?

Answers

On sοlving the prοvided questiοn we can say that equatiοn Miles already ran -target distance = Miles still tο run 60 - 24 = 36 miles left tο run

What is equatiοn?  

A mathematical equatiοn is a fοrmula that links twο statements and uses the equals sign (=) tο indicate equality. In algebra, an equatiοn is a statement that demοnstrates the equality οf twο mathematical expressiοns.

We may begin by subtracting the miles Mοna has already ran frοm her οbjective distance tο get the number οf miles she must run each day tο accοmplish her gοal:

Miles already ran - target distance = Miles still tο run

60 - 24 = 36 miles left tο run

Since there are three days left in the week, Mοna must run:

36 miles ÷ 3 days = 12 miles per day

On a number line, we may begin with 24 and add 12 fοr each remaining day:

24 ------ 36 ------ 48 ------ 60

Tο represent this οn a number line, we can start at 24 and add 12 fοr each remaining day:

|---|---|---|---|---|---|---|

0 5 10 15 20 25 30 35 40

^ ^ ^

Day 5 6 7

To know more about number line visit:

https://brainly.com/question/11064576

#SPJ1

Students must purchase a ticket to attend the show. The committee wants to have $425 remaining from ticket sales, after paying for all expenses.
Based on this information, what should be the price of each ticket? Explain your reasoning. PLS HURRY

Answers

The cost of an individual ticket is contingent upon the overall expenditure of the performance and the quantity of tickets purchased.

What is ticket selling?

Ticket sales refer to the purchase of one or multiple admission passes to an occasion, which may be facilitated either through the Event Management System, online, mobile app, third-party voucher providers (such as Groupon), or physically, among other options.

Based on the fact that there is no much details on the ticket sales, it present ]an obstacle in knowing the number cost of individual tickets. To determine the cost of each ticket, we need to have the total expenditure or the quantity of tickets sold.

Learn more about ticket selling from

https://brainly.com/question/30496695

#SPJ1

50 Points! Multiple choice algebra graphing question. Which square root function is represented by the graph? Photo attached. Thank you!

Answers

A square root function that is represented by the graph include the following: A. f(x) = √(4x + 8).

What is a square root function?

In Mathematics and Geometry, a square root function can be defined as a type of function that typically has this form f(x) = √x, which basically represent the parent square root function i.e f(x) = √x.

In Mathematics and Geometry, a horizontal translation to the left is modeled by this mathematical equation g(x) = f(x + N) while a vertical translation to the positive y-direction (downward) is modeled by this mathematical equation g(x) = f(x) - N.

Where:

N represents an integer.g(x) and f(x) represent functions.

Therefore, the required square root function is given by;

f(x) = √(4x + 8)

Read more on translation here: brainly.com/question/4521517

#SPJ1

Solve the following equation for θ on the interval [0,2π).
2sec(θ)+3=0
Round your answers to the nearest thousandth of a radian.
there should be 2 answers

Answers

We can start by isolating the secant term and then taking the inverse cosine of both sides. Remember that the inverse cosine function only gives results on the interval [0,π], so we will need to adjust our solutions accordingly:

2sec(θ) + 3 = 0

2sec(θ) = -3

sec(θ) = -3/2

cos(θ) = -2/3

θ = ±cos⁻¹(-2/3)

Using a calculator, we find:

θ ≈ 2.302, 4.981 radians

What is the value of 0 in the equation?

However, we need to adjust these solutions to be within the interval [0,2π). Since cos(x) = cos(2π - x), we can add to the negative solution to get an equivalent positive solution:

θ ≈ 2.302, 7.438 radians

Rounding to the nearest thousandth, we get:

θ ≈ 2.302, 7.438 radians

Learn more about radian from

https://brainly.com/question/22760025

#SPJ1

You have $2000 to deposit into a bank account that earns simple interest. Use interest rates at local banks to find how much interest different accounts earn after four years. Compare the different accounts and decide which account you should use. Consider other fees associated with each account when making your decision.

Answers

Bank B would be the best option since it offers a higher interest rate than Bank A and does not have any fees.

How to compare different bank accounts?

To compare different bank accounts, we need to know the interest rates offered by each bank. Let's assume that we have three local banks, each with a different interest rate: Bank A offers 2% interest, Bank B offers 2.5% interest, and Bank C offers 3% interest.

Using the formula for simple interest, we can calculate the amount of interest earned on the initial deposit of $2000 over four years for each bank:

Bank A: 2% interest per year

After four years, the interest earned is:

$2000 x 2% x 4 = $160

Bank B: 2.5% interest per year

After four years, the interest earned is:

$2000 x 2.5% x 4 = $200

Bank C: 3% interest per year

After four years, the interest earned is:

$2000 x 3% x 4 = $240

So, Bank C offers the highest interest rate and would earn the most interest over the four-year period. However, we also need to consider any fees associated with each account. Let's assume that Bank A and Bank B have no fees, but Bank C charges a $50 annual fee.

Bank A: Total amount after four years = $2000 + $160 = $2160

Bank B: Total amount after four years = $2000 + $200 = $2200

Bank C: Total amount after four years = $2000 + $240 - $50 x 4 = $1960

Even though Bank C offers the highest interest rate, the annual fee significantly reduces the total amount earned over four years. Therefore, Bank B would be the best option since it offers a higher interest rate than Bank A and does not have any fees.

Learn about interest rate here https://brainly.com/question/25793394

#SPJ1

Is this quadrilateral a parallelogram?

Answers

Answer:

Yes it is a parallelogram because opposite sides of this quadrilateral are equal.

A right triangle has sides 3 and 5 . Use the Pythagorean Theorem to find the length of the hypotenuse.

Answers

The Pythagorean Theorem states that in a right triangle, the square of the length of the hypotenuse is equal to the sum of the squares of the lengths of the other two sides. In this case, c^2 = 3^2 + 5^2 = 9 + 25 = 34, so c = √34.
The answer is 6 cm. Step by step explanation : the formula for pythagorean theorem is square root of a^2+b^2 which gives you c^2 so if you place the 3 and the 5 in the square root and square then you will get 9 + 25 in a square root which would give you 34 if you add the two and the square root of 34 is 5.8(to 1 decimal place) it’s honestly up to you if you would like to round up to a whole number which would be 6 cm. Hope this helped and I hope I didn’t complicate it too much

Q13

Steve receives £200 each month from his parents as an allowance.
20% of this amount is spent on his football magazine subscription, and the rest he spends on football stickers,
sweets, and fizzy drinks in the ratio of 5: 2: 1.
How much does Steve spend on football stickers?

Answers

Step-by-step explanation:

Steve spends 20% of £200 = 0.2 * £200 = £40 on his football magazine subscription.

The remaining amount he spends on football stickers, sweets, and fizzy drinks in the ratio of 5:2:1, which means that for every 5 parts he spends on football stickers, he spends 2 parts on sweets and 1 part on fizzy drinks.

So, let's call the amount he spends on football stickers "5x". Then, the amount he spends on sweets is "2x" and the amount he spends on fizzy drinks is "x".

The total amount he spends on these items is:

5x + 2x + x = 8x

We know that he has £200 - £40 = £160 left after paying for his magazine subscription. So, we can set up an equation:

8x = £160

Solving for x, we get:

x = £20

Therefore, Steve spends:

5x = 5 * £20 = £100

on football stickers.

Can you mark me asbrainliest if you appreciate my effort just for favor. Thank you^^

Answer:

5 : 2 : 1 = 100 : 40 : 20

Step-by-step explanation:

10% = 20 (200÷10=20)

20% = 40 (double both sides)

take away £40 from £200 = £160

£160 is left over after buying the magazine subscription

______________________________

add the radio all together = 5:2:1 = 8

divide 8 by 160 = £20

That now means 1 = 20

5 : 2 : 1

we already found 1, now to find 2 and 5

you times 2

1 = 20

2 = 40

and then do the same for 5

1 = 20

5 = 100

To know this is right, 100 + 40 + 20 = 160 the original number for the money left over.

Richard is saving money for a down payment on a car. He opens a savings account at his local bank and deposits $500. He models his savings plan with the equation y = 200x + 500 based on his current monthly savings rate.

Answers

Here the correct statements can be

The y-intercept represents D) the starting amount in his savings account.

The slope represents C) the change in the dependent variable with respect to the independent variable.

What is meant by y-intercept?

The y-intercept is the point where a line intersects the y-axis on a graph. It represents the value of the dependent variable when the independent variable is zero.

What is meant by the term slope?

The slope is a measure of the steepness of a line on a graph. It is calculated by dividing the change in the y-coordinate by the change in the x-coordinate between two points on the line.

According to the information

The y-intercept of the linear model y = 200x + 500 represents the starting amount in Richard's savings account.

The slope of the linear model y = 200x + 500 represents the change in the dependent variable (y, the savings amount) with respect to the independent variable (x, the number of months). Specifically, the slope of 200 means that for every additional month, Richard's savings will increase by $200.

To know more about slope visit

brainly.com/question/19131126

#SPJ1

21) Kinzang is 1.49 m tall. He was 0.53 m at birth. How much did he grow?​

Answers

Kinzang grew 0.96 meters.

We have,

To find how much Kinzang grew, we need to subtract his height at birth from his current height:

Height Kinzang grew = Current height - Height at birth

Height Kinzang grew = 1.49 m - 0.53 m = 0.96 m

Therefore,

Kinzang grew 0.96 meters.

Learn more about expressions here:

https://brainly.com/question/3118662

#SPJ1

The manager of the San Marcos Sliders baseball team keeps track of the number of runs the team scores each season. This histogram shows the distribution of runs scored per game this season.
Complete the sentences.
The distribution of runs scored is best described as
. So, the mean absolute deviation
an appropriate measure of variation.

Answers

It shtbe noted that 68% of the data should fall in the interval from 616 to 756.

How to explain the information

z score for values:

z = (X-µ)/σ = (616-686)/70 = -1

z = (X-µ)/σ = (756-686)/70 = 1

Approximately 68% of the data should fall in the interval from 616 to 756.

b) B. 71% of the data falls in the interval from 616 to 756. The estimate is very close to the value predicted by the Empirical Rule.

c) (µ - 2*σ, µ + 2*σ)

(686 - 2*70, 686 + 2*70)

(546, 826)

You expect to find about 95% of the teams between the two values 546 and 826.

Learn more about data on

https://brainly.com/question/26711803

#SPJ1

Which equations represent circles that have a diameter of 12 units and a center that lies on the y-axis? Select two options. x2 + (y – 3)2 = 36 x2 + (y – 5)2 = 6 (x – 4)² + y² = 36 (x + 6)² + y² = 144 x2 + (y + 8)2 = 36

Answers

The correct option are - A. x²  + (y – 3)²  = 36 and E. x²  + (y + 8)²  = 36 , for the equation of circle.

Explain about the equation of circle:

The standard form equation for just a circle, which is (x - s)² + (y - t)² = r², is used to formulate our equation if we assume a generic centre point like (s,t)and r is the radius of circle.

By simply expanding basic binomial squares in their regular form and merging like words, circles can likewise be presented in expanded form.

Now for the given data:

Diameter of circle = 12 unitsRadius r = 12/2 = 6 unitscentre lies on the y axis, (s,t) = (0,t)

Put the values in the standard form of the circle:

(x - s)² + (y - t)² = r²,

(x - 0)² + (y - t)² = 6²,

x² + (y - t)² = 36

Where 't' can be any integer.

Thus, the options that satisfy the stated conditions are-

A. x²  + (y – 3)²  = 36 : here t = 3

and E. x²  + (y + 8)²  = 36 ; here t = - 8.

Know more about the equation of circle.

https://brainly.com/question/1506955

#SPJ1

Correct question:

Which equations represent circles that have a diameter of 12 units and a center that lies on the y-axis?

Select two options.

A. x²  + (y – 3)²  = 36

B. x²  + (y – 5)²  = 6

C. (x – 4)² + y² = 36

D. (x + 6)² + y² = 144

E. x²  + (y + 8)²  = 36

Other Questions
What is Peroneus Tertius (Insertion and Innervation)? The following argument is an example of which logical fallacy? Dogs are the best pets because they are loyal and follow directions. You are a loyal friend. You must own a dog. a. Circular reasoning b. Non sequitur c. Slippery slope d. Red herring Italians were bitter and angry over the peace agreement, too. They were upset when they did not receive __ after being on the winning side of World W Why do fast-food restaurants tend to cluster in the same location Suppose that two fast-food franchises, A and B, initially locate near each other in the center of town. The state recently opened a new exit ramp from the nearby interstate highway. Although both restaurants are operating profit- ably, the restaurant owners calculate that by relocating near the intestate highway they will continue to receive some in-town business, but will benefit from sales to drivers using the exit as a rest stop. There are four possible strategy profiles: {Exit ramp, Exit ramp), (Exit ramp, Downtown}, {Downtown, Exit ramp), and (Downtown, Downtown). The respective pay- offs for (A, B), which are in millions of dollars are: (0.15, 0.15), (1.0, 0.1), (0.1, 1.0), and (0.25, 0.15). a. Does either player have a strictly-dominant strategy? b. Does this game have a Nash equilibrium strategy profile? c. Are these restaurants better off by forming a coalition? Is this coalition stable? In a command economy, producers have all the choice about what goods and services to produce. True/ False Compute the first three natural frequencies and the corresponding mode shapes of the transverse vibrations of a uniform beam of rectangular cross section (100 mm times 300 mm) with 1 = 2 m, E = 20. 5 times 10^10 N/m^2, and p = 7. 83 times 10^3 kg/m^3 for the following cases: (a) when both ends are simply supported; (b) when both ends are built-in (clamped); when one end is fixed and the other end is free; and (d) when both ends are free. Plot the mode shapes A weather balloon used by meteorologists is made of a flexible bag that allows the gas inside freely expand. If a weather balloon containing 25.0 m^3 of helium gas is released from sea level, what is the volume of gas when the balloon reaches a height of 2100 m, where the pressure is 0.8210^5 PA? Assume the temperature is unchanged. LOTS OF POINTSQuestion refers to the excerpt below."If our great corporations would more scrupulously observe their legal limitations and duties, they would have less cause to complain of the unlawful limitations of their rights or of violent interference with their operations. The community that by concert, open or secret, among its citizens denies to a portion of its members their plain rights under the law has severed the only safe bond of social order and prosperity. Those who use unlawful methods, if moved by no higher motive than the selfishness that prompted them, may well stop and inquire what is to be the end of this."Benjamin Harrison, from his Inaugural Address, March 4, 1889 Harrison's speech and its emphasis on the effects of corporate abuse would be a criticism of government regulation of business laissez-faire economic policy socialist ideology grassroots politics The nurse is closely monitoring a child who is unconscious after a fall and notices that the child suddenly has a fixed and dilated pupil. The nurse should interpret this as:a. eye trauma.b. neurosurgical emergency.c. severe brainstem damage.d. indication of brain death. Mildred starts to agree with her husband in part 2 of the novel In July 1964, an elderly woman was mugged in Costa Mesa, California. In the vicinity of the crime a tall, bearded man sat waiting in a yellow car. Shortly after the crime was committed, a young, tall woman, wearing her blond hair in a ponytail, was seen running from the scene of the crime and getting into the car, which sped off. The police broadcast a description of the suspected muggers. Soon afterward, a couple fitting the description was arrested and convicted of the crime. Although the evidence in the case was largely circumstantial, the two people arrested were nonetheless convicted of the crime. The prosecutor based his entire case on basic probability theory, showing the unlikeness of another couple being in that area while having all the same characteristics that the elderly woman described. The following probabilities were used. Characteristic Assumed probability Drives yellow car 1 out of 12 Man over 6 feet tall 1 out of 10 Man wearing tennis shoes 1 out of 4 Man with beard 1 out of 11 Woman with blond hair 1 out of 3 Woman with hair in a ponytail 1 out of 13 Woman over 6 feet tall 1 out of 100 Compute the probability of another couple being in that area with the same characteristics. Would you use the addition or multiplication rule? Why? Are the characteristics independent or dependent? How are the computations affected by the assumption of independence or dependence? Should any court case be based solely on probabilities? Would you convict the couple who was arrested even if there were no eyewitnesses? Comment on why in todays justice system no person can be convicted solely on the results of probabilities. In actuality, arent most court cases based on uncalculated probabilities? 42. When the Japanese questioned the salaries of American CEOs during a time when many companies were having difficulty and laying off workers, they were making their judgments based upon:a. utilitarian justiceb. distributive justicec. procedural justiced. rule-based justice some languages are genetically related to each other. a _________ is the ancestral language from which related languages have developed. What is one of the main functions of Supreme Court clerks? McCarthyism: Joseph McCarthy, U.S. Senator, made unsubstantiated claims that more than 200 "card carrying" members of the Communist party had infiltrated the US government. He had no proof PLEASE ANSWER!!!! 40 POINTS!!!!!!!From previous steps, we know 2.0 mol P4O10 can form 8.0 mol H3PO4 and 8.0 mol H2O can form 5.3 mol H3PO4.How many moles of H3PO4 can form during the reaction?mol H3PO4 Please help I have no idea how to do this Primarily used to monitor devices on a network, such as monitoring a router's state remotely is called? What is the error in the solution of the proportion shown at the right?A. The second line should be 2x + 18 = 3x making the final answer 18 = xB. The second line should be 2x + 27 = 3x making the final answer 27= xC. The second line should be 3x + 9 = 2x making the final answer 9 = xD. There is no error Which is an autotroph: sycamore tree, zooplankton, centipede, or antelope?